You are on page 1of 52

 

INSIGHTS  MOCK  TEST  SERIES  2015:  TEST  –  31  SOLUTIONS  


 
SCD. Unfortunately, most people with SCD are
1. Solution: a) either too old for a transplant or don’t have a
relative who is a good enough genetic match for
Lichens are composite, symbiotic organisms them to act as a donor. A well-matched donor
made up from members of as many as three is needed to have the best chance for a
kingdoms. The dominant partner is a fungus. successful transplant.
Fungi are incapable of making their own food.
They usually provide for themselves as http://www.thehindu.com/todays-paper/tp-
parasites or decomposers. national/sickle-cell-anaemia-stalks-adivasis-
says-study/article7431619.ece
The lichen fungi (kingdom Fungi) cultivate
partners that manufacture food by
photosynthesis. Sometimes the partners are
algae (kingdom Protista), other times 3. Solution: a)
cyanobacteria (kingdom Monera), formerly Currency swap agreements exist to assist
called blue-green algae. Some enterprising countries during periods of tight liquidity or
fungi exploit both at once. balance of payments and liquidity crises.
Pioneer species are hardy species which are the Under this agreement, Sri Lanka can draw a
first to colonize previously disrupted or maximum of $1.1 billion for a period of up to
damaged ecosystems, beginning a chain of six months.
ecological succession that ultimately leads to a
more biodiverse steady-state ecosystem. The agreement is in addition to the existing
Examples can be phytoplankton, zooplankton, framework on currency swap arrangement for
lichens, mosses etc. the SAARC member countries, an arrangement
by which SAARC members can draw currency
$100 million-$400 million, with a total limit of
2. Solution: c) $2 billion, from an RBI financing facility set up
for this purpose.
Sickle-cell disease (SCD), also known as sickle-
cell anaemia (SCA) and drepanocytosis, is a
hereditary blood disorder, characterized by an 4. Solution: d)
abnormality in the oxygen-carrying
haemoglobin molecule in red blood cells. Zoos and botanical gardens are the most
conventional methods of ex situ conservation,
“Inherited” means that the disease is passed by all of which house whole, protected specimens
genes from parents to their children. SCD is for breeding and reintroduction into the wild
not contagious. A person cannot catch it, like a when necessary and possible.
cold or infection, from someone else.
Endangered plants may also be preserved in
At the present time, hematopoietic stem cell part through seed banks or germ plasm banks.
transplantation (HSCT) is the only cure for The term seed bank sometimes refers to a
http://www.insightsonindia.com                                                            INSIGHTS   Page  1  
 
 
INSIGHTS  MOCK  TEST  SERIES  2015:  TEST  –  31  SOLUTIONS  
 
cryogenic laboratory facility in which the seeds
of certain species can be preserved for up to a
century or more without losing their fertility.

It can also be used to refer to a special type of


arboretum where seeds are harvested and the
crop is rotated. For plants that cannot be
preserved in seed banks, the only other option
for preserving germ plasm is in-vitro storage,
where cuttings of plants are kept under strict
conditions in glass tubes and vessels.

Also read the drawbacks of ex-situ


conservation
https://en.wikipedia.org/wiki/Ex_situ_conser
vation#Drawbacks

6. Solution: a)
5. Solution: c)
Species like Blue whale and Whooping crane
Under the Wildlife (Protection) Act, the state
are highly susceptible to extinction because of
governments have to notify the list of core and
the reasons stated in statement 1 and 2.
buffer areas of tiger reserves in their territory.
Island species are also likely to be extinct
Core zone (critical tiger habitats) is where
because of their narrow range of distribution.
tigers usually rest, reside, feed and breed.
Moreover, species which are higher in food
Buffer zones are areas that lie in the periphery
chain have greater chances to be extinct, not
of the core zone.
the ones that lie at the bottom of the food
Buffer zones constitute the fringe areas i.e. the chain.
outside boundary of tiger reserves up to 10
kms.
7. Solution: d)

Mercury is used in laboratories for making


thermometers, barometers, diffusion pumps,
and many other instruments. It is used for
mercury switches and other electrical
apparatus. It is used as an electrode in some
types of electrolysis and for making batteries
(mercury cells).

http://www.insightsonindia.com                                                            INSIGHTS   Page  2  


 
 
INSIGHTS  MOCK  TEST  SERIES  2015:  TEST  –  31  SOLUTIONS  
 
Potassium dichromate has uses in photography 9. Solution: d)
and in photographic screen printing, where it is
used as an oxidizing agent together with a Social forestry is defined as “Forestry outside
strong mineral acid. the conventional forests which primarily aim at
providing continuous flow of goods and
The light sensitive silver halides, silver services for the benefit of people. This
chloride, silver bromide and silver iodide, are definition implies that the production of forest
used to make photographic film and goods for the needs of the local people is Social
photographic paper. forestry. Thus, social forestry aims at growing
forests of the choice of the local population

On the other hand, agro-forestry includes a


variety of land uses where woody species are
grown in combination with crops. For instance
crops can be grown in between rows of saal,
8. Solution: b) teak etc.

Apart from being used in manures seaweeds or


marine algae are a food source for humans
especially in East Asia, it is most commonly 10. Solution: d)
associated with Japanese food. Seaweeds also About 84% of total evaporation comes from the
are used to make a number of food additives oceans. They receive 77% of the total
such as alginates and carrageenan which is precipitation.
used in cooking and baking as a vegetarian
alternative to gelatine. 16% evaporation comes from land. Land
receives 24% precipitation. There is a gain of
Many seaweeds are used as medicine. Alginates 7% in land.
are used in wound dressings and in the
production of dental moulds and agar is used This excess is returned by surface run-off and
very widely in Microbiology to help grow sub-surface flows to the ocean again.
bacterial cultures.
Thus the water cycle maintains equilibrium.
Seaweeds are ingredients in toothpaste,
cosmetics and paints and are used in industrial
products such as paper coatings, adhesives,
dyes, gels, explosives and many more.

Much of the oil and natural gas we use today


formed from seaweeds which partially 11. Solution: b)
decomposed on the sea floor many millions of
Production functions include producing furits,
years ago.
woods, resins, alkaloids, medicines etc.

http://www.insightsonindia.com                                                            INSIGHTS   Page  3  


 
 
INSIGHTS  MOCK  TEST  SERIES  2015:  TEST  –  31  SOLUTIONS  
 
Protective functions include conservation of Psedomonas is a denitrifying bacterium. It
soil and water, prevention of drought, shelter converts nitrate back to Nitrogen for release in
against cold, wind and radiation, noise etc. the atmosphere.

Regulative functions help improve atmosphere


and temperature conditions.
14. Solution: a)
The above functions tell us that the economical
value of forests is very high and they play a very § It is a symbiotic relation of fungi with
important role in maintaining the biological roots of vascular plants.
cycle on earth. § Nearly all plants on earth rely on
mycorrhizal fungi for nutrients
and moisture. Many plants are
12. Solution: c) extremely dependent and can struggle to
survive without the beneficial fungi
Mineralization in soil science is decomposition (grapes and roses are examples).
or oxidation of the chemical compounds
(minerals) in organic matter into plant- § Plants and mycorrhizal fungi
accessible forms. Mineralization is the opposite operate as a single working unit in
of immobilization. nature. The plant performs
photosynthesis and other above-ground
Immobilization (or demineralisation) in soil functions, and the fungi handle
science is the conversion of inorganic underground nutrition-gathering and
compounds to organic compounds by micro- protect the roots. It is not normal to
organisms or plants, by which it is prevented grow plants without mycorrhizae
from being accessible to plants. Immobilization on the roots – this is often the
is the opposite of mineralization. cause of disease and insect
problems!

13. Solution: c) § Other benefits include enhanced rooting


of cuttings, increased root generation,
Bacteria that change nitrogen gas from the increased drought resistance, increased
atmosphere into solid nitrogen usable by plants salt tolerance, reduced transplant shock,
are called nitrogen-fixing bacteria. These and enhancement of other valuable
bacteria are found both in the soil and in organisms in the soil.
symbiotic relationships with plants.

Rhizobium is not a free-living bacterium. It is


tied to the host legume plant in a symbiotic
relationship.

http://www.insightsonindia.com                                                            INSIGHTS   Page  4  


 
 
INSIGHTS  MOCK  TEST  SERIES  2015:  TEST  –  31  SOLUTIONS  
 
Phosphates move quickly through plants and
animals; however, the processes that move
them through the soil or ocean are very slow,
making the phosphorus cycle overall one of the
slowest biogeochemical cycles. Rain water
contains phosphorus and nitrogen from air
pollution.

Initially, phosphate weathers from rocks and


minerals, the most common mineral being
apatite. Overall small losses occur in terrestrial
environments by leaching and erosion, through
the action of rain. In soil, phosphate is
absorbed on iron oxides, aluminium
hydroxides, clay surfaces, and organic matter
particles, and becomes incorporated
(immobilized or fixed). Plants and fungi can
also be active in making P soluble.
15. Solution: d)

It has been explained beautifully here


http://www.majordifferences.com/2014/03/di 17. Solution: b)
fference-between-c3-and-c4- 50% of available solar radiation is
plants.html#.VavrYaSqqko photosynthetically active radiation. Rest of the
C4 plants have a competitive advantage over energy is deflected by the atmosphere.
plants possessing the more common C3 carbon You also need to know that visible light is the
fixation pathway under conditions of drought, only band of light on the spectrum to be
high temperatures, and nitrogen or CO2 considered photosynthetically active. It has the
limitation. perfect amount of energy to excite the electrons
needed to start photosynthesis and not damage
DNA or break bonds.
16. Solution: b)
Ultraviolet cannot be used for photosynthesis
Unlike many other biogeochemical cycles, the because it has too much energy. This energy
atmosphere does not play a significant role in breaks the bonds in molecules and can destroy
the movement of phosphorus, because DNA and other important structures in
phosphorus and phosphorus-based compounds organism.
are usually solids at the typical ranges of
temperature and pressure found on Earth. P
cannot be found in the air as a gas. 18.Solution: d)
http://www.insightsonindia.com                                                            INSIGHTS   Page  5  
 
 
INSIGHTS  MOCK  TEST  SERIES  2015:  TEST  –  31  SOLUTIONS  
 
Species that have strong effects on the that India and Korea will strengthen
composition of communities are called their historic connection by enhancing
Keystone Species. linkages of Korean people with Ayodhya.

Example: Starfish, Humming Bird, Sea Otter, • A decision was also taken to upgrade the
African elephants, Beaver, Flying Fox, Prairie monument for Queen Suriratna, also
dogs. know as Hur Hwang-ok, in Ayodhya as a
joint project between the two countries.
In addition to keystone species, there are other
categories of species that are crucial to their
ecosystem's survival.
20. Solution: d)
Critical link Species are species which play an
important role in supporting network species http://www.thehindu.com/todays-
by functioning as pollinators, nutrient paper/heres-the-first-look-of-
circulators or absorbers. There are a large amaravati/article7439266.ece
number of critical link species in any http://www.deccanchronicle.com/150405/nati
ecosystem. on-current-affairs/article/all-you-need-know-
The species which are found most abundantly about-andhra-pradesh%E2%80%99s-capital-
in ecotone boundary are known as edge city
species.

Ecotone is defined as the place or area, where 21. Solution: c)


two major communities meet and blend
together. It consists of species of both the The Betwa or Betravati is a river in Northern
communities. India, and a tributary of the Yamuna. Also
known as the Vetravati, the Betwa rises in the
Vindhya Range just north of Hoshangabad in
19. Solution: a) Madhya Pradesh and flows north-east through
Madhya Pradesh and Orchha to Uttar Pradesh.
http://indpaedia.com/ind/index.php/The_Kar Nearly half of its course, which is not
ak_clan_of_Queen_Hwang-ok/_Suri_Ratna navigable, runs over the Malwa Plateau.

• India has begun the work of tracing its The Ken River is one of the major rivers of the
“shared heritage” with Korea using the Bundelkhand region of central India, and flows
legendary Queen Suriratna, a princess through two states, Madhya Pradesh and Uttar
from Ayodhya who travelled to the Pradesh. It is a tributary of the Yamuna.
country to marry King Kim Suro in 48
AD, as a pivot. http://www.thehindu.com/todays-paper/tp-
national/interlinking-of-rivers-vital-for-water-
• During Prime Minister Narendra Modi’s food-security-minister/article7439174.ece
visit to Seoul in May, it was announced
http://www.insightsonindia.com                                                            INSIGHTS   Page  6  
 
 
INSIGHTS  MOCK  TEST  SERIES  2015:  TEST  –  31  SOLUTIONS  
 
populations of other five union territories are
too small to have any representative in the
Rajya Sabha.

22. Solution: c) 24. Solution: c)


Though the President of India is not a member Under the Constitution, a person shall be
of either House of Parliament and does not sit disqualified for being elected as a Member of
in the Parliament to attend its meetings, he is Parliament:
an integral part of the Parliament.
• if he holds any office of profit under the
This is because a bill passed by both the
Union or state government (except that
Houses of Parliament cannot become law
of a minister or any other office
without the President’s assent. He also
exempted by Parliament).
performs certain functions relating to the
• if he is of unsound mind and stands so
proceedings of the Parliament, for example, he
declared by a court.
summons and prorogues both the Houses,
• if he is an undischarged insolvent.
dissolves the Lok Sabha, addresses both the
Houses, issues ordinances when they are not in • if he voluntary (not expelled) gives
session, and so on. up the membership of the political party
on whose ticket he is elected to the
House
• if he is not a citizen of India or has
23. Solution: a)
voluntarily acquired the citizenship of a
The representatives of states in the Rajya foreign state or is under any
Sabha are elected by the elected members of acknowledgement of allegiance to a
state legislative assemblies. The election is held foreign state; and
in accordance with the system of proportional • if he is so disqualified under any law
representation by means of the single made by Parliament.
transferable vote. • He must not be a director or managing
agent nor hold an office of profit in a
The representatives of each union territory in corporation in which the government
the Rajya Sabha are indirectly elected by has at least 25 per cent share.
members of an electroral college specially • He must not have been dismissed from
constituted for the purpose. This election is government service for corruption or
also held in accordance with the system of disloyalty to the State.
proportional representation by means of the
single transferable vote. Out of the seven union
territories, only two (Delhi and Puducherry)
have representation in Rajya Sabha. The 25. Solution: a)

http://www.insightsonindia.com                                                            INSIGHTS   Page  7  


 
 
INSIGHTS  MOCK  TEST  SERIES  2015:  TEST  –  31  SOLUTIONS  
 
The Speaker is elected by the Lok Sabha from Speaker is the final interpreter of the
amongst its members (as soon as may be, after provisions of (a) the Constitution of India, (b)
its first sitting). Whenever the office of the the Rules of Procedure and Conduct of
Speaker falls vacant, the Lok Sabha elects Business of Lok Sabha, and (c) the
another member to fill the vacancy. The date of parliamentary precedents, within the House.
election of the Speaker is fixed by the Powers of Chairman, Rajya Sabha are the
President. same.

Usually, the Speaker remains in office during Moreover, they derive their powers and duties
the life of the Lok Sabha. However, he has to from three sources, that is, the Constitution of
vacate his office earlier in any of the following India, the Rules of Procedure and Conduct of
three cases: Business of Lok Sabha, and Parliamentary
Conventions (residuary powers that are
o if he ceases to be a member of the unwritten or unspecified in the Rules).
Lok Sabha;
o if he resigns by writing to the
Deputy Speaker; and
o if he is removed by a resolution
passed by a majority of all the 27. Solution: c)
members of the Lok Sabha.
The institutions of Speaker and Deputy
Such a resolution can be moved only after Speaker originated in India in 1921 under the
giving 14 days advance notice. provisions of the Government of India Act of
When a resolution for the removal of the 1919 (Montague–Chelmsford Reforms). At that
Speaker is under consideration of the House, time, the Speaker and the Deputy Speaker were
he cannot preside at the sitting of the House, called the President and Deputy President
though he may be present. However, he can respectively and the same nomenclature
speak and take part in the proceedings of the continued till 1947.
House at such a time and vote in the first Before 1921, the Governor- General of India
instance, though not in the case of an equality used to preside over the meetings of the
of votes. Central Legislative Council. In 1921, the
It should be noted here that, whenever the Lok Frederick Whyte and Sachidanand Sinha were
Sabha is dissolved, the Speaker does not vacate appointed by the Governor-General of India as
his office and continues till the newly- elected the first Speaker and the first Deputy Speaker
Lok Sabha meets. (respectively) of the central legislative
assembly. In 1925, Vithalbhai J. Patel became
the first Indian and the first elected Speaker of
the central legislative assembly.
26. Solution: d)

http://www.insightsonindia.com                                                            INSIGHTS   Page  8  


 
 
INSIGHTS  MOCK  TEST  SERIES  2015:  TEST  –  31  SOLUTIONS  
 
The Government of India Act of 1935 changed members are from Lok Sabha only. The Rajya
the nomenclatures of President and Deputy Sabha has no representation in this committee.
President of the Central Legislative Assembly These members are elected by the Lok Sabha
to the Speaker and Deputy Speaker every year from amongst its members,
respectively. according to the principles of proportional
representation by means of a single
transferable vote.
28. Solution: c) Thus, all parties get due representation in it.
The concept of ‘equality before law’ is an The term of office is one year. A minister
element of the concept of ‘Rule of Law’, cannot be elected as a member of the
propounded by A.V. Dicey, the British jurist. committee. The chairman of the committee is
His concept has the following three elements or appointed by the Speaker from amongst its
aspects: members and he is invariably from the ruling
party.
(i) Absence of arbitrary power, that is, no man
can be punished except for a breach of law.

(ii) Equality before the law, that is, equal 30. Solution: b)
subjection of all citizens (rich or poor, high or Financial committees of Estimates, PSUs, and
low, official or non-official) to the ordinary law Public Accounts keep track of expenditure, its
of the land administered by the ordinary law usefulness and performance of the
courts. administrative spending.
(iii) The primacy of the rights of the individual, Some laws need to be filled with administrative
that is, the constitution is the result of the details by the instrument of delegated
rights of the individual as defined and enforced legislation. The committee on delegated
by the courts of law rather than the legislation scrutinizes such rules and
constitution being the source of the individual regulations formed by the executive or
rights. bureaucracy.
The first and the second elements are Zero hour and question hour are used to raise
applicable to the Indian System and not the any matter in Lok Sabha. The administration
third one. In the Indian System, the can be held accountable.
constitution is the source of the individual
rights.

31. Solution: d)

29. Solution: a) Article 75 of the Constitution says that the


council of ministers shall be collectively
Originally, it had 25 members but in 1956 its responsible to the Lok Sabha. It means that the
membership was raised to 30. All the thirty
http://www.insightsonindia.com                                                            INSIGHTS   Page  9  
 
 
INSIGHTS  MOCK  TEST  SERIES  2015:  TEST  –  31  SOLUTIONS  
 
ministry stays in office so long as it enjoys fundamental rights shall be void. In other
confidence of the majority of the members of words, it expressively provides for the doctrine
the Lok Sabha. of judicial review.

In other words, the Lok Sabha can remove the This “power” (not source of power, read
ministry from office by passing a no-confidence carefully) has been conferred on the Supreme
motion. The motion needs the support of 50 Court (Article 32) and the high courts (Article
members to be admitted. 226) that can declare a law unconstitutional
and invalid on the ground of contravention of
any of the Fundamental Rights.
32. Solution: d) Thus, not only a legislation but any of the
Popular sovereignty is a basic idea of above can be challenged in the courts as
democracy. Popular sovereignty means that the violating a Fundamental Right and hence, can
people are the ultimate source of the authority be declared as void.
of their government.

Popular sovereignty means that democratic 34. Solution: d)


government is by the people and for the
people—for the benefit of the people, not for UPSC only certifies eligible candidates.
the benefit of those who govern in their name. Appointment is done by DoPT (Government of
India).
Government in a democracy is the servant of
the people; it is not their master. Therefore, Service Conditions of AIS are determined by
those who govern are public servants—they the Parliament.
hold public office only to serve the
people, not to serve themselves. In a Members of AIS cannot be removed by an
democracy, political authority flows from the authority subordinate to the one that appointed
people to the government—not from it. President appoints members of AIS. So, they
government to the People. cannot be removed by the PM.

Popular sovereignty means that the


government can only exercise authority if it has 35. Solution: a)
been given permission to do so by the
People. Therefore, popular sovereignty limits The Appropriation Bill becomes the
the powers of government. Appropriation Act after it is assented to by the
President. This act authorises (or legalises) the
payments from the Consolidated Fund of India.
33. Solution: c) This means that the government cannot
withdraw money from the Consolidated Fund
Article 13 declares that all laws that are of India till the enactment of the appropriation
inconsistent with or in derogation of any of the
http://www.insightsonindia.com                                                            INSIGHTS   Page  10  
 
 
INSIGHTS  MOCK  TEST  SERIES  2015:  TEST  –  31  SOLUTIONS  
 
bill. This takes time and usually goes on till the Every department and official is given
end of April. information about the amount of funds they
have and they have to perform within it - if it
But the government needs money to carry on goes overboard then they are accountable.
its normal activities after 31 March (the end of These funds are passed by the
the financial year). To overcome this functional Parliament/Legislature through voting. The
difficulty, the Constitution has authorised the auditing and reporting work is ex post facto as
Lok Sabha to make any grant in advance in it is not easy to get the finance back once the
respect to the estimated expenditure for a part Finance Ministry sanctions funds to the
of the financial year, pending the completion of departments.
the voting of the demands for grants and the
enactment of the appropriation bill. So, ultimately Parliament has to hold the
executive financially accountable.
This provision is known as the ‘vote on
account’. It is passed (or granted) after the
general discussion on budget is over. It is
generally granted for two months for an 38. Solution: d)
amount equivalent to one-sixth of the total The International North–South Transport
estimate. Corridor is the ship, rail, and road route for
moving freight between India, Russia, Iran,
Europe and Central Asia. The route primarily
36. Solution: d) involves moving freight from India, Iran,
Azerbaijan and Russia via ship, rail and road.
The Finance Ministry of India presents the
Economic Survey in the parliament every year, The objective of the corridor is to increase
just before the Union Budget. It is the trade connectivity between major cities such as
ministry's view on the annual economic Mumbai, Moscow, Tehran, Baku, Bandar
development of the country. Abbas, Astrakhan, Bandar Anzali and etc.

Economic Survey reviews the developments in Dry runs of two routes were conducted in 2014,
the Indian economy over the previous 12 the first was Mumbai to Baku via Bandar Abbas
months, summarizes the performance on major and the second was Mumbai to Astrakhan via
development programs, and highlights the Bandar Abbas, Tehran and Bandar Anzali.
policy initiatives of the government and the
prospects of the economy in the short to
medium term. This document is presented to
both houses of Parliament during the Budget
Session (not along with the budget).

37. Solution: c) 39. Solution: d)

http://www.insightsonindia.com                                                            INSIGHTS   Page  11  


 
 
INSIGHTS  MOCK  TEST  SERIES  2015:  TEST  –  31  SOLUTIONS  
 
British cosmologist Steven Hawking on As powerful regional kingdoms emerged in
Monday launched the biggest-ever search for various parts of India, Delhi could no longer
intelligent life in the universe in a 10-year, function as an effective centre.
$100-million (143-million-euro) project to scan
the heavens.
41. Solution: c)
• The Breakthrough Listen project,
backed by Russian Silicon Valley When Alivardi Khan died in 1756,
entrepreneur Yuri Milner, will be the Sirajuddaulah became the nawab of Bengal.
most powerful, comprehensive and The Company was worried about his power and
intensive scientific search ever keen on a puppet ruler who would willingly
undertaken for signs of extra-terrestrial give trade concessions and other privileges. So
intelligent life. it tried, though without success, to help one of
• The project will use some of the biggest Sirajuddaulah’s rivals become the nawab.
telescopes on Earth, searching far
deeper into the universe than before for An infuriated Sirajuddaulah asked the
radio spectrum and laser signals. Company to stop meddling in the political
• The initiative is allied with the affairs of his dominion, stop fortification, and
Breakthrough Message project, an pay the revenues. After negotiations failed, the
international competition to create Nawab marched with 30,000 soldiers to the
digital messages that represent English factory at Kassimbazar, captured the
humanity. Company officials, locked the warehouse,
disarmed all Englishmen, and blockaded
There is no commitment to send any messages English ships. Then he marched to Calcutta to
into space, and the project should spark establish control over the Company’s fort there.
discussion about whether humans should be
sending messages at all out into the void. On hearing the news of the fall of Calcutta,
Company officials in Madras sent forces under
the command of Robert Clive, reinforced by
naval fleets. Prolonged negotiations with the
40. Solution: a)
Nawab followed. Finally, in 1757, Robert Clive
Aurangzeb was the last of the powerful Mughal led the Company’s army against Sirajuddaulah
rulers. at Plassey.

He established control over a very large part of


the territory that is now known as India. After
42. Solution: d)
his death in 1707, many Mughal governors
(subadars) and big zamindars began asserting This page deals brilliantly with Gandhian
their authority and establishing regional ideology of democracy and state.
kingdoms.

http://www.insightsonindia.com                                                            INSIGHTS   Page  12  


 
 
INSIGHTS  MOCK  TEST  SERIES  2015:  TEST  –  31  SOLUTIONS  
 
http://www.mkgandhi.org/articles/indian_de from membership, shall fulfill in good
mocracy.html faith the obligations assumed by
them in accordance with the present
Mahatma Gandhi’s imagination of the Charter.
democracy -fully encircled with non-violence -
exists in no nation of the world as up to now. • All Members shall settle their
Democracy of his imagination happens to be international disputes by peaceful
one, which does not have any provision of means in such a manner that
punishment and even an organization like international peace and security, and
‘State’ happens to be obsolete in it. This is justice, are not endangered.
because Mahatma Gandhi holds, “…State is
symbolical of centralized and organized • All Members shall refrain in their
violence.” As non-violence is connected with international relations from the threat
human soul, man can be non-violent whereas or use of force against the territorial
in opposition to it, “… State is a soul-less integrity or political independence of
machine. On this accord, it is impossible to get any state, or in any other manner
rid of violence. Its very existence depends upon inconsistent with the Purposes of the
violence.” Philosophy of Mahatma Gandhi, United Nations.
non-violence should be admitted as invariable
• All Members shall give the United
part of our life and it is on the basis of this
Nations every assistance in any action it
dictum that modem polities must operate.
takes in accordance with the present
While in the present day democracy, there is a Charter, and shall refrain from giving
great deal of centralization and inequality. In a assistance to any state against which the
stateless democracy there is decentralization United Nations is taking preventive or
and equality. enforcement action.

• The Organization shall ensure that


states which are not Members of the
United Nations act in accordance with
43. Solution: d) these Principles so far as may be
The Organization and its Members, in pursuit necessary for the maintenance of
of the Purposes stated in Article 1, shall act in international peace and security.
accordance with the following Principles. • Nothing contained in the present
• The Organization is based on the Charter shall authorize the United
principle of the sovereign equality of Nations to intervene in matters which
all its Members. are essentially within the domestic
jurisdiction of any state or shall require
• All Members, in order to ensure to all of the Members to submit such matters to
them the rights and benefits resulting settlement under the present Charter;

http://www.insightsonindia.com                                                            INSIGHTS   Page  13  


 
 
INSIGHTS  MOCK  TEST  SERIES  2015:  TEST  –  31  SOLUTIONS  
 
but this principle shall not prejudice the Suffering can be ended by knowing the four
application of enforcement measures noble truths and following the eightfold path.
under Chapter Vll. They essentially centre on ending ignorance by
following dharma, right meditation,
contemplation etc.
44. Solution: c) You can understand their meanings here
The Wadiyar dynasty was an Indian Hindu http://www.buddhanet.net/e-
dynasty that ruled the Kingdom of Mysore learning/intro_bud.htm
from 1399 to 1947.

It was in news recently due to the coronation of 46. Solution: c)


the new prince.
Dana is a virtue and duty in Jainism. It is
http://www.thehindu.com/news/national/kar considered an act of compassion, and must be
nataka/yaduveer-gopal-raj-urs-is-new-heir-of- done with no desire for material gain.
mysuru-royal-family/article6886404.ece
Four types of Dana are discussed in the texts of
http://indianexpress.com/article/india/india- Jainism: Ahara-dana (donation of food),
others/prince-of-mysore-srikantadatta- Ausadha-dana (donation of medicine), Jnana-
narasimharaja-wodeyar-passes-away/ dana (donation of knowledge) and and Abhaya-
Four wars were fought with Mysore (1767-69, dana (giving of protection or freedom from
1780-84, 1790-92 and 1799). Only in the last – fear, asylum to someone under threat).
the Battle of Seringapatam – did the Company Dāna is one of ten means to gain positive
ultimately win a victory. Tipu Sultan was killed karma, in the soteriological theories of Jainism.
defending his capital Seringapatam, Mysore Medieval era texts of Jainism dedicate a
was placed under the former ruling dynasty of substantial portion of their discussions to the
the Wodeyars and a subsidiary alliance was need and virtue of Dāna.
imposed on the state.

47. Solution: b)
45. Solution: d)
Large Kingdoms and stately cities made their
The Buddha told us, "The nature of Human life appearance in the later Vedic Period. In
is suffering". (He did not say all living being Taittariya Brahmana you will notice the theory
suffer).
of the divine origin of kingship.
This suffering has a case which is ignorance of
The government machinery became more
what ‘is’.
elaborate than before , as a sequel to the
growth of the power of the king . New civil
functionaries, besides the only civil functionary
http://www.insightsonindia.com                                                            INSIGHTS   Page  14  
 
 
INSIGHTS  MOCK  TEST  SERIES  2015:  TEST  –  31  SOLUTIONS  
 
of the Rigvedic Period the purohita came into Vaishnavism (Vaisnava dharma) is one of the
existence. These were : the Bhagadudha ( major branches of Hinduism along with
Collector of taxes), the Suta/ Sarathi (the Shaivism, Smartism, and Shaktism. It is
Royalherald or Charioteer ), the Khasttri focused on the veneration of Vishnu.
(Chamberlain), The Akshavapa (Courier). Vaishnavites, or the followers of the Vishnu,
lead a way of life promoting the central
The military officials of the Rigvedic times, the importance of Vishnu and his ten avatars.
Senani (the head of the village ) continued to
function. Vaishnavism flourished in predominantly
Shaivite South India during the seventh to
tenth centuries CE and is still commonplace,
48. Solution: b) especially in Tamil Nadu, as a result of the
twelve Alvars, saints who spread the sect to the
Gandhi’s entry into public life began with the common people with their devotional hymns.
‘Satyagraha’ in Champaron district of Bihar in
1917. He could mobilize the peasants of this In later years, Vaishnava practices increased in
district against the exploitation of European popularity due to the influence of sages like
indigo planters. In 1918, Gandhiji led a “no tax Ramanujacharya, Madhvacharya,
campaign” at Khera in Gujarat where the Nimbarkacharya, Vallabhacharya, Vedanta
peasants were not able to pay the revenue due Desika, Manavala Mamunigal, Surdas,
to famine. Tulsidas, eknath, Tyagaraja, and many others.
But not everyone adopted and embraced
But, after the Jalianwala Bagh tragedy, the Vaishnavism
government expressed no sign of regret but
went ahead with more repression.

Mahatma Gandhi was shocked and suspended 50. Solution: d)


the ‘Satyagraha’ declaring it as a “Himalayan http://www.thehindu.com/todays-paper/tp-
Blunder”. opinion/a-battle-without-
It was because he had asked those people to winners/article7449218.ece
pursue non-violence who could not afford to be The novel Madhorubagan, written by Perumal
non-violent. The Satyagraha movement failed Murugan published four years ago, deals with a
in attaining its object as the government did ritual that has been practiced in Tiruchengode
not withdraw the Rowlatt Act. However, it was a century ago. As per the ritual, a women could
the first experiment of non-violence of go with any man on the night of the festival. A
Gandhiji in Indian politics. child born out of such a relation was treated as
gift of God. In the novel, a couple is childless
and the wife wants to take part in the ritual.
49. Solution: c) The husband is opposed to it but is forced to be
a silent witness when the wife choses to go her

http://www.insightsonindia.com                                                            INSIGHTS   Page  15  


 
 
INSIGHTS  MOCK  TEST  SERIES  2015:  TEST  –  31  SOLUTIONS  
 
way. The protest by some Hindu bodies is In the Bachan singh case it gave the rarest of
against the "sexual permissiveness' and rate doctrine according to which death penalty
blasphemy as shown in the book. should be given only in the extreme cases and
where life sentence should not at all be
appropriate.

Last year, while handing down its Shatrughan


Chauhan vs Union of India ruling, the
Supreme Court spelt out clear guidelines on the
legal rights of prisoners on death row.

51. Solution: a)

The spacecraft also made several predictions 53. Solution: d)


about the planet which should be read here. It
is quite contrary to what was understood till Agreement for establishing NDB was signed
date about Pluto. during the 6th BRICS Summit being held in
Fortaleza, Brazil in April, 2014. It was formally
http://www.thehindu.com/todays-paper/tp- launched at the 7th BRICS summit held in Ufa,
opinion/historic-flyby/article7449216.ece Russia in July 2015.

The Kuiper belt sometimes called the Purpose: To fund infrastructure projects in the
Edgeworth–Kuiper belt, is a region of the Solar emerging economies. It is seen as an
System beyond the planets, extending from the alternative institute to west dominated World
orbit of Neptune (at 30 AU) to approximately Bank and the International Monetary Fund
50 AU from the Sun. (IMF).

It is similar to the asteroid belt, but it is far Capital: It will have initial capital of US 50
larger—20 times as wide and 20 to 200 times billion dollars and will be raised to US 100
as massive. billion dollars within the next couple of years.

Each member’s role: They will have an equal


say in the bank’s management, regardless of
52. Solution: b)
GDP size and contribute an equal share in
The Supreme Court in Mithu vs State of Punjab establishing a startup capital.
struck down Section 303 of the Indian Penal
President: Eminent banker Kundapur Vaman
Code, which provided for a mandatory death
Kamath from India is President of Bank for the
sentence for offenders serving a life sentence.
first five years i.e. till 2020.
http://www.thehindu.com/opinion/editorial/d
eath-sentences-trial-and-
error/article7448579.ece 54. Solution: d)

http://www.insightsonindia.com                                                            INSIGHTS   Page  16  


 
 
INSIGHTS  MOCK  TEST  SERIES  2015:  TEST  –  31  SOLUTIONS  
 
According to the OECD ‘Government at a (b) any information which he knows to be false,
Glance 2015’ report, trust represents the but for the purpose of causing annoyance,
confidence of citizens and businesses in inconvenience, danger, obstruction, insult,
government to do what is right and perceived injury, criminal intimidation, enmity, hatred or
as fair. ill will, persistently by making use of such
computer resource or a communication device,
Changes in trust levels could be affected by
many factors, including the economic outlook,
political changes such as elections or other
major events such as disasters or major 56. Solution: d)
scandals including corruption cases. Moreover, In its 50-year history, Japan has been the only
expectations of citizens could grow at a faster Asian country to hold it with India being
pace than government responses. bestowed the honor to host the 12th such
This is the only survey that collects data on the International symposium now.
issue of trust in governments. It is a sampling The Themes for the Symposium include
survey of 1,000 citizens in each country Antarctica and Supercontinent Evolution,
conducted by World Poll, which has been doing Antarctic Surface Processes, Landscapes, and
it since 2005. Links with Cryosphere and Climate, Antarctic
Solid Earth Structure and Interactions with the
Cryosphere: Antarctica, the Southern Ocean,
55. Solution: d) and Evolution of Climate and the Global
Cryosphere, amongst others.
IPC Section 499/500 was in news recently.

The Centre has told the Supreme Court that


defamation should remain a penal offence in 57.Solution: d)
India as the defamer may be too poor to • The deal puts strict limits on Iran’s
compensate the victim. It has denied that nuclear activities for at least a decade
criminal defamation had any chilling effect on and calls for stringent U.N. oversight,
free speech. The IPC under Section 499/500 with world powers hoping this will make
criminalizes defamatory speech. This means any dash to make an atomic bomb
that a person can be imprisoned for a virtually impossible.
maximum period of 2 years, if found guilty. • In return, Iran will get sanctions relief
although the measures can “snap back”
According to the Section 66A of the IT act, any
into place if there are any violations.
person who sends, by means of a computer
• The international arms embargo against
resource or a communication device,-
Iran will remain for five years but
(a) any information that is grossly offensive or
deliveries would be possible with special
has menacing character; or
permission of the U.N. Security Council.
Iran has accepted allowing the U.N.

http://www.insightsonindia.com                                                            INSIGHTS   Page  17  


 
 
INSIGHTS  MOCK  TEST  SERIES  2015:  TEST  –  31  SOLUTIONS  
 
atomic watchdog tightly-controlled The main aim of the DAC is to fast-track
“managed access” to military bases. procurement process of the armed forces by
• Iran will slash by around two-thirds the optimally utilising the available budget.
number of centrifuges from around
19,000 to 6,104.
• The deal caps uranium enrichment at
3.67% and limits the stockpile to 300 kg,
all for 15 years.
• Iran will be required to ship spent fuel 59. Solution: c)
out of the country forever, as well as
allow inspectors from the IAEA
inspectors certain access in perpetuity.
Heightened inspections, including
tracking uranium mining and
monitoring the production and storage
of centrifuges, will last for up to 20
years.

58. Solution: b)

DAC was set up in October 2001 following


recommendations from Group of Ministers
(GoMs) on ‘Reforming the National Security
System.’

The need for DAC was felt post-Kargil conflict


and this high-level body is chaired by the
Defence Minister.

Other members include: Minister of State for


60. Solution: a)
Defence, Chief of Army Staff, Chief of Naval
• It will comprise members of the Centre
Staff, Chief of Air Staff, Defence Secretary,
and states to promote India’s overseas
Secretary Defence Research & Development,
shipments.
Secretary Defence Production, Chief of
Integrated Staff Committees (HQ IDS), • The council will be chaired by the Union
Director General (Acquisition) and Deputy Commerce and Industry Minister and
Chief of Integrated Defence Staff. secretaries of key ministries and state
ministers will be the members.

http://www.insightsonindia.com                                                            INSIGHTS   Page  18  


 
 
INSIGHTS  MOCK  TEST  SERIES  2015:  TEST  –  31  SOLUTIONS  
 
• The main objective of the council will be • The Union government has agreed to
facilitating trade from states in a bid to bring it under the Modified Industrial
boost the country’s exports and Infrastructure Upgradation Scheme
rationalising non-essential imports.. (MIIUS).

• The move comes amid declining exports


which fell 5.45 per cent to $25.1 billion
62. Solution: c)
in June, while imports fell 13.46 per cent
to $35.4 billion. The Supreme Court recently directed the
government to have a re-look at the drug
• The essential issues that will be taken up
pricing policy to help make life-saving
with the state officials include
medicines affordable for the common man.
infrastructure bottlenecks in terms of
road connectivity, power supply, law • The government had approved the
and order and overall governance; National Pharmaceutical Pricing Policy
regulatory environment; and local (NPPP) in 2012.
taxation-related matters particularly • This policy at bringing 348 essential
where refunds are involved. drugs under price control and also lead
http://indianexpress.com/article/business/bus to reduction in prices. With this, the
iness-others/with-aim-to-promote-exports- Govt would control prices of 348
govt-to-set-up-trading-facilitation-council/ essential drugs.
• The policy debars the companies from
using the Wholesale Price Index (WPI)
to increase the prices of the essential
61. Solution: a)
medicines on their own each year. Thus,
In a major fillip to industrial infrastructure in the companies had to seek approval
the Palakkad region, the Union government’s from the National Pharmaceutical
Department of Industrial Policy and Promotion Pricing Authority whenever they wanted
(DIPP) has approved a proposal from the to increase the prices of the items
Kerala Industrial Infrastructure Development covered under the Drug Price Control
Corporation (Kinfra) to set up the country’s Order.
first defence industrial park at Ottappalam. • It covers only 348 drugs covered under
National list of life saving medicines.
• The proposed park will be established as
part of the Make in India, Make in
Kerala project
63. Solution: d)
• It will have modern common
infrastructure facilities aimed at The GI tag ensures that none other than those
attracting component manufacturers in registered as authorised users (or at least those
the defence industry. residing inside the geographic territory) are

http://www.insightsonindia.com                                                            INSIGHTS   Page  19  


 
 
INSIGHTS  MOCK  TEST  SERIES  2015:  TEST  –  31  SOLUTIONS  
 
allowed to use the popular product name. • The scientific programs of this natural
Darjeeling tea became the first GI tagged polar laboratory are designed and
product in India, in 2004-05, since then by approved by the Scientific Committee on
September 2010, 184 had been added to the Antarctic Research (SCAR) which has a
list. These are listed below. President and two to three vice-
presidents and a small secretariat
https://en.wikipedia.org/wiki/List_of_Geogra
located at Cambridge, England.
phical_Indications_in_India
• The science research programs are
Just scan through the list. Its a long one. Be proposed and co-ordinated by three
aware of popular names. standing scientific groups (SSG’s) on
Geosciences (GS), life sciences (LS) and
physical sciences (PS) each having three
chief officers.
64. Solution: d)
• India currently has two permanent
The “Jansankhya Sthirata Kosh” (JSK) stations, Maitri at Schirmacher oasis
(National Population Stabilisation Fund) has and Bharati in the Larsemann Hill area
been registered as an autonomous Society located ~1000km east of Maitri. The
established under the Societies Registration first Indian station, Dakshin Gangotri,
Act of 1860. located on shelf ice is now buried and
lost.
• The Union Health Minister heads the
General Body of JSK and the Ministries
of Health and Family Welfare, Women
66. Solution: d)
and Child Development, Department of
School Education & Literacy, Rural • This Bill amends the principal Act
Development, Planning Commission are passed in 2013.
represented by their Secretaries on the
General Body of JSK. • The Bill enables the government to
• All State Governments are members of exempt five categories of projects from
JSK. the requirements of: (i) social impact
assessment, (ii) restrictions on
acquisition of multi-cropped land, and
(iii) consent for private projects and
65. Solution: c)
public private partnerships (PPPs)
• Antarctica is recognized as a heritage of
projects.
mankind and as mandated by the
provisions of the Antarctic treaty, signed • The five categories of projects are: (i)
by 29 consultative parties, no defence, (ii) rural infrastructure, (iii)
commercial activity is permitted in affordable housing, (iv) industrial
Antarctica and its environment and all corridors, and (v) infrastructure
claims to its territory are frozen.
http://www.insightsonindia.com                                                            INSIGHTS   Page  20  
 
 
INSIGHTS  MOCK  TEST  SERIES  2015:  TEST  –  31  SOLUTIONS  
 
including PPPs where government owns
the land.

• The Act would apply retrospectively, if


an award had been made five years
earlier and compensation had not been
paid or possession not taken. The Bill
exempts any period when a court has
given a stay on the acquisition while
computing the five year period.

• The Act deemed the head of a


government department guilty for an
offence by the department. The Bill
removes this, and adds the requirement
of prior sanction to prosecute a
government employee.

In front of the chariot two elephants are


67. Solution: c)
positioned as if they are pulling the chariot. In
The stone chariot located inside the campus of fact these elephants where brought from
Vittala temple is almost an iconic structure of elsewhere and positioned here at a later stage.
Hampi. The Stone Chariot at the Vijaya Vittala Originally two horses were carved in that
temple complex in Hampi, Karnataka will now position. The tails and the rear legs of the
adorn the new Rs. 10 note. horses can be still seen just behind these
elephant sculptures.
An image of Garuda (eagle god) was originally
enshrined within its sanctum. Garuda,
according to the Hindu mythology, is the
vehicle of lord Vishnu. Thus the Garuda shrine
facing the temple’s sanctum is symbolic. 68. Solution: d)

In reality this stone shrine was built with many It will be adorn a Rs. 50 note as per a
giant granite blocks. The joints are smartly decision taken recently by the government.
hidden in the carvings and other decorative
features that adorn the Stone Chariot. The Sun Temple is a 13th-century Sun Temple at
chariot is built on a rectangular platform of a Konark in Odisha, India. It is believed that the
feet or so high. temple was built by king Narasimhadeva I of
Eastern Ganga Dynasty around 1250 CE. The
temple is in the shape of a gigantic chariot with
elaborately carved stone wheels, pillars and
http://www.insightsonindia.com                                                            INSIGHTS   Page  21  
 
 
INSIGHTS  MOCK  TEST  SERIES  2015:  TEST  –  31  SOLUTIONS  
 
walls. A major part of the structure is now in
ruins. The temple is a UNESCO World Heritage
Site. It is also featured on NDTV's list of Seven
Wonders of India and Times of India's list of
Seven Wonders of India.

The temple was originally built at the mouth of


the river Chandrabhaga, but the waterline has
receded since then. The temple has been built
in the form of a giant ornamented chariot of
the Sun god, Surya.

69. Solution: b)

Goa churches were in news as they will adorn


the new Rs. 500 note. Amongst them most
prominent is this church.
It has been choosen to adorn new 1000
The Basilica of Bom Jesus or Borea Jezuchi
currecny notes in India.
Bajilika is located in Goa, India, and is a
UNESCO World Heritage Site. The basilica It is a beautiful painting of Ajanta caves. This
holds the mortal remains of St. Francis Xavier. segment from Gardner’s Art through the Ages:
The church is located in Old Goa, which was Non-Western Perspectives (2009) describes
the capital of Goa in the early days of the scene shown:
Portuguese rule.
The bodhisattva Padmapani sits among a
'Bom Jesus' (literally, 'Good (or Holy) Jesus') is crowd of devotees, both princesses and
the name used for the Ecce Homo in the commoners. With long, dark hair handing
countries of Portuguese colonization. The down below a jeweled crown, he stands
Jesuit church is India’s first minor basilica, and holding his attribute, a blue lotus flower, in his
is considered to be one of the best examples of right hand. […] The artist has carefully
baroque architecture in India. considered the placement of the painting in the
cave. The bodhisattva gazes downward at
70. Solution: a)
worshipers passing through the entrance to
the shrine on their way to the rock-cut Buddha
image in a cell at the back of the cave.

Also go through this beautiful description of


Ajanta caves.

http://www.insightsonindia.com                                                            INSIGHTS   Page  22  


 
 
INSIGHTS  MOCK  TEST  SERIES  2015:  TEST  –  31  SOLUTIONS  
 
https://www.khanacademy.org/humanities/ar and bursts, bringing rainfall as high as 75
t-asia/south-asia/buddhist-art2/a/the-caves- millimeters per hour.
of-ajanta

72. Solution: d)
71. Solution: c)
The move came after a petitioner said the non-
A cloudburst is an extreme amount of native eucalyptus tree was responsible for the
precipitation, sometimes accompanied by hail lowering groundwater levels in the region.
and thunder that normally lasts no longer than
a few minutes but is capable of creating flood • A eucalyptus tree consumes 90 litres of
conditions. water a day

A cloudburst can suddenly dump 72,300 tons • During summers and times of
of water over one acre. However, cloudbursts drought, its roots can go down up to 30ft
are infrequent as they occur only via • It was introduced in Karnataka in 1960s.
orographic lift or occasionally when a warm air In the 1970’s, eucalyptus plantations
parcel mixes with cooler air, resulting in were spread across 2.1 lakh hectares.
sudden condensation.

Cloud burst is a situation when the inter-


molecular forces between the H2O molecules 73. Solution: a)
get very high due to the rapid decrease in the
§ The GSLV-III or Geosynchronous
temperature or excess of electrostatic induction
Satellite Launch Vehicle Mark III, is a
in the clouds causing the lighting to remain
launch vehicle developed by the Indian
inside the cloud only, which causes hyperactive
Space Research Organization.
energy inside the cloud. The water molecules
get denser and denser and get condensed but § GSLV Mk III is conceived and designed
do not leave the cloud due to excess of to make ISRO fully self reliant in
electroforces. launching heavier communication
satellites of INSAT-4 class, which weigh
As the water concentration get higher and
4500 to 5000 kg.
higher and so the weigh gets heavier the water
no longer is able to maintain force with the § It would also enhance the capability of
clouds and so they fall and it precipitates. the country to be a competitive player in
the multimillion dollar commercial
In the Indian subcontinent, a cloudburst
launch market. The vehicle envisages
usually occurs when a monsoon cloud drifts
multi-mission launch capability for
northwards, from the Bay of Bengal or Arabian
GTO, LEO, Polar and intermediate
Sea across the plains, then onto the Himalaya
circular orbits.

http://www.insightsonindia.com                                                            INSIGHTS   Page  23  


 
 
INSIGHTS  MOCK  TEST  SERIES  2015:  TEST  –  31  SOLUTIONS  
 
of India, an advisory group and a
committee on the same.
• At present, the renewable energy sector
is governed by the Electricity Act, 2003,
which is also undergoing amendments.
• The policy would enable a supportive
system for growth of the sector.
74. Solution: d)
• The various segments which are the
Indian space industry experts recently opined focus of the policy are: Renewable
that there is a need to have a space law to energy resource assessment, technical
protect sovereign, public or commercial and safety standards, monitoring and
interests in India. verification, manufacturing and skill
development and data management.
This is to ensure that space assets and • Through a separate law, the ministry of
applications are used for the right causes. new and renewable energy (MNRE)
There is no single space law in India. would get freedom to execute projects
and not depend on other ministries and
Currently, space activities are guided by a
departments for necessary clearances,
handful of international space agreements, the
said officials.
Constitution, national laws, the Satellite
• The law also aims to set up dedicated
Communications (SatCom) Policy of 2000 and
renewable electricity investment zones.
the revised Remote sensing policy or 2011.
The law makes it clear who will finance,
India is among the five countries that do not who will plan and monitor and what
have a space law; while 15 others including the support will come from where.
US, Russia, Japan, China, Kazakhstan and
Ukraine, have laws based broadly on the Outer
Space Treaty of 1967. 76. Solution: b)

The security legislation proposed by Japanese


Prime Minister Shinzo Abe’s administration
75.Solution: c)
moved a step closer to becoming law when
The government has drafted the National Parliament’s lower house approved it. The bills,
Renewable Energy Bill, 2015 which aims to which seek to rewrite the country’s post-War
consolidate the renewable energy sector and pacifist security policy, are now before the
give it an institutional structure. upper house.

• After it is passed by Parliament it would Over a period of seven decades, Japan’s


enable a National Renewable Energy security policy, shaped under a war-renouncing
Policy, Renewable Energy Corporation Constitution following the misadventures of

http://www.insightsonindia.com                                                            INSIGHTS   Page  24  


 
 
INSIGHTS  MOCK  TEST  SERIES  2015:  TEST  –  31  SOLUTIONS  
 
the imperial regime, has been focussed on self- forms parallel to the boundary, to the
defence. mountain range, and to the trench. Powerful
earthquakes shake a wide area on both sides of
But the present bills seek to replace the self- the boundary.
defence doctrine with “collective self-defence”,
that would allow Japan to send troops abroad This usually happens with an oceanic plate
to rescue allies under attack. This big shift in being subducted under a continental plate.
approach makes the legislation controversial
and unpopular. Mountains are more likely to be formed when
two continental plates converge.
For reasons refer this article:

http://www.thehindu.com/opinion/editorial/j
apans-security-legislation-approved-by-lower-
house/article7444346.ece

79. Solution: c)
77. Solution: c) Rivers are cheap means of transport, compared
Rocks that carry organic matter are broken in to roads and railway. But most rivers in Africa
sediments of different sizes. are not navigable due to water falls, weeds and
being seasonal.
A river then carries it from higher altitude in
form of sediment and then deposits it layer Africa need to combine forces to utilise this
wise from high-lying areas (coarse) to low-lying rivers for irrigation, hydro electric power and
areas (fine clay having organic matter). combat the big problem of transport by
overcoming weeds and waterfalls.
These sediments get compressed and cemented
together under high temperature and pressure Navigation can be done only in Zambeji and
to become sedimentary rocks. And fossil fuel at Nile river.
an even greater temperature and pressure
applied over a large period of time.
80. Solution: d)

Heating the atmosphere is secondary. Visible


78. Solution: d) light from the Sun penetrates the atmosphere
When two plates come together, it is known as and strikes objects on the Earth's surface.
a convergent boundary. The impact of the two These objects absorb that energy and re-emit
colliding plates buckles the edge of one or both that energy as infra red energy that heads back
plates up into a rugged mountain range, and out into space.
sometimes bends the other down into a deep
seafloor trench. A chain of volcanoes often

http://www.insightsonindia.com                                                            INSIGHTS   Page  25  


 
 
INSIGHTS  MOCK  TEST  SERIES  2015:  TEST  –  31  SOLUTIONS  
 
But, the atmosphere contains gases like water By the rising and falling action of tides, they
vapor and water droplet clouds that absorb that take back all the mud deposited by incoming
infra red energy. So the energy is re-emitted rivers to the sea. Otherwise the water
back and forth. This heats the atmosphere. availability for the ships would have reduced.
Basically the atmosphere is heated by the
Earth's surface which is heated by the Sun.
83. Solution: a)

81.Solution: d) Murmansk in the Arctic and the Baltic ports


near St. Petersburg are both important for
Katabatic wind, also called downslope wind, or Russian access to the Atlantic. Despite St.
gravity wind, wind that blows down a slope Petersburg being much further south, it is
because of gravity. It occurs at night, when the surrounded by ice for a month or more each
highlands radiate heat and are cooled. year, while Murmansk is always ice free.

The air in contact with these highlands is thus Warm currents increase the temperature of the
also cooled, and it becomes denser than the air sea water. By the action of land and sea breezes
at the same elevation but away from the slope; the local area temperature is also increased.
it therefore begins to flow downhill. This Hence, the ports remain ice-free.
process is most pronounced in calm air because
winds mix the air and prevent cold pockets
from forming. 84. Solution: a)
When a katabatic wind is warmed by Most large objects in orbit around the Sun lie
compression during its descent into denser air, near the plane of Earth's orbit, known as the
it is called a foehn. A large-scale katabatic wind ecliptic. The planets are very close to the
that descends too rapidly to warm up is called a ecliptic, whereas comets and Kuiper belt
fall wind. In areas where fall winds occur, objects are frequently at significantly greater
homes and orchards are situated on hill slopes angles to it.
above the lowlands where the cold air
accumulates. All the planets and most other objects orbit the
Sun in the same direction that the Sun is
rotating (counter-clockwise, as viewed from
82. Solution: a) above Earth's north pole). There are
exceptions, such as Halley's Comet.
Statement 4 is not true because indentation of
coasts is a long term natural phenomenon. It is Venus and Uranus rotate in the opposite
caused by erosion of coasts and a host of other direction to what they revolve.
factors. It is not caused by tides.

85. Solution: b)

http://www.insightsonindia.com                                                            INSIGHTS   Page  26  


 
 
INSIGHTS  MOCK  TEST  SERIES  2015:  TEST  –  31  SOLUTIONS  
 
If the Moon didn't spin at all, then eventually it o Angle of incidence
would show its far side to the Earth while o Duration of sunshine
moving around our planet in orbit. o Solar constant
o Distance between the earth and
However, since the rotational period is exactly the sun
the same as the orbital period, the same o Transparency of the atmosphere.
portion of the Moon's sphere is always facing
the Earth. The vertical rays of the sun heat the minimum
possible area, but on the contrary, the oblique
It can be understood in detail here rays are spread over a relatively larger area, so
that the amount of area over which the
http://www.moonconnection.com/moon- available solar energy has to be distributed in
same-side.phtml increased and the energy per unit area on the
earth's surface is decreased.

In addition, the oblique rays have to traverse a


larger distance through the atmosphere before
they strike the surface of the earth. The longer
their path, the larger the amount of energy lost
86. Solution: d) by various processes of reflection, absorption,
and scattering, etc.
Prime Meridian and all other meridians are
semi-circles, not circles.
88. Solution: d)
Time zone of a place is decided only by its
latitude. This is because the earth rotates The decision is arrived after a joint meeting
around the Sun from West to East. between the Secretary of the Union Finance
Ministry, the Chief General Manager of the
The rotation causes the Sun pass overhead
Currency Management Department, Reserve
from one place from the east to west. Only
Bank of India, and the Director-General of
latitude makes the Sun’s entry timing different
Archaeological Survey of India (ASI).
in the regions.
Recently they agreed to put new images on the
currency notes. For e.g. Stone chariot of Vittal,
87. Solution: a) Hampi is to be put on a Rs. 10 note.

The actual amount of insolation received at a


place on the earth varies according to the
conditions of the atmosphere as well as the
seasons. The following astronomical and 89. Solution: a)
geographical factors govern the amount of
insolation received at any point on the earth's
surface:
http://www.insightsonindia.com                                                            INSIGHTS   Page  27  
 
 
INSIGHTS  MOCK  TEST  SERIES  2015:  TEST  –  31  SOLUTIONS  
 
A rise in the purchasing power (income) of the GDP grows when the present market value of
consumer can sometimes induce the consumer this year’s produce is more than that of last
to reduce the consumption of a good. year.

The demand for such a good can be inversely or But growth in total market value can also be
positively related to its price depending on the because of high inflation and no rise in
relative strengths of these two opposing effects. production.
If the substitution effect is stronger than the
income effect, the demand for the good and the So, real GDP indicator is used to determine
price of the good would still be inversely whether there has been an actual growth in
related. production.

However, if the income effect is stronger than At a constant base price (2011-12 year) in India,
the substitution effect, the demand for the good increase in total value is calculated. So, real
would be positively related to its price. Such a GDP will only increase if there has been even
good is called a Giffen good. slight growth in production irrespective of
inflation or deflation in the economy.

90. Solution: b)

Consumer welfare refers to the individual


benefits derived from the consumption of
goods and services. In theory, individual 92. Solution: a)
welfare is defined by an individual's own
assessment of his/her satisfaction, given prices To understand the concept properly refer to the
and income. Exact measurement of consumer article
welfare therefore requires information about
http://www.ehow.com/about_6605945_intere
individual preferences.
st-rate-employment.html
The central idea in consumer welfare is to
In brief, low interest rates entice greater
enhance consumer surplus which is the
investments in the economy for they promise
difference between what people prefer to pay
better returns on projects.
and what they actually pay. The greater the
difference, higher is the surplus. It means that More investment leads to more employment
the market is allocating goods most efficiently creation. However, this relationship need not
(at competitive prices) to people. hold necessarily in the long run.

91. Solution: b) 93. Solution: b)

http://www.insightsonindia.com                                                            INSIGHTS   Page  28  


 
 
INSIGHTS  MOCK  TEST  SERIES  2015:  TEST  –  31  SOLUTIONS  
 
The Reserve Bank of India was established on
April 1, 1935 in accordance with the provisions
of the Reserve Bank of India Act, 1934. 95. Solution: d)

The Central Office of the Reserve Bank was Liquidity trap is a situation when expansionary
initially established in Calcutta but was monetary policy (increase in money supply)
permanently moved to Mumbai in 1937. The does not increase the interest rate, income and
Central Office is where the Governor sits and hence does not stimulate economic growth.
where policies are formulated. It is a situation in which the general public is
Though originally privately owned, since prepared to hold on to whatever amount of
nationalisation in 1949, the Reserve Bank is money is supplied, at a given rate of interest.
fully owned by the Government of India. They do so because of the fear of adverse events
like deflation, war.
The constitution makes no mention of the RBI.
There is a liquidity trap at short term zero
percent interest rate. When interest rate is
zero, public would not want to hold any bond,
94. Solution: a) since money, which also pays zero percent
It refers to the process by which the RBI takes interest, has the advantage of being usable in
away money from the banking system to transactions.
neutralise the fresh money that enters the
system from foreign capital inflows. Hence, if the interest is zero, an increase in
quantity of money cannot not induce anyone to
It does so by selling or buying government buy bonds and thereby reduce the interest on
bonds from public. bonds below zero.

If it does not do so, money supply in the


economy will shoot up in a very short period of
time playing havoc with the interest rates and 96. Solution: c)
inflation in the economy. The objectives of the Public Distribution
Refer to the article to better understand System is to ensure food security and poverty
sterilization alleviation by making available the essential
commodities, specially food grains at an
http://www.thehindubusinessline.com/iw/200 affordable and uniform price at the door steps
3/02/02/stories/2003020201001400.htm of the consumers.

The department of Food and Civil supplies


arranges procurement of PDS items like rice,
wheat/atta, levy sugar, iodised salt
(procurement not being done now), Kerosene

http://www.insightsonindia.com                                                            INSIGHTS   Page  29  


 
 
INSIGHTS  MOCK  TEST  SERIES  2015:  TEST  –  31  SOLUTIONS  
 
oil, LPG under various Govt. of India scheme employment and higher production of goods
under Public Distribution System. and services.

A strong industry favours greater growth in the


services as well as primary sector.
97. Solution: a)
But, if an economy becomes highly dependent
Devaluation of currency means more of that on the primary sector, it will not be able to
currency can be purchased for any given produce adequate amount of goods and
foreign currency. services for its population.
This means more of domestic goods can be On a different note, financial risks have become
purchased from abroad following a fall in a part of the global economic environment.
currency’s value. Better policies and institutional coordination
In other words, since our currency becomes amongst global and national monetary and
more affordable to the foreigners, so do the fiscal authorities is required to rationalize this
goods that are priced in them. risk.

98. Solution: c) 100. Solution: a)

When economy is operating at a much lower


level then there is scope for immediate increase https://en.wikipedia.org/wiki/The_Energy_an
in production of goods following an increase in d_Resources_Institute
demand.

If economy is already operating at its peak,


then industries will find it difficult to produce
more goods even by adding extra labour since
the productivity of labour will be limited given
the limit of capital goods (machines).  

Therefore, only when the industry can produce


more (supply) as per the demand, a greater
money supply will not lead to inflation.

99. Solution: a)

Economic development in the modern world is


fundamentally based on industrialization
which leads to generation of gainful
http://www.insightsonindia.com                                                            INSIGHTS   Page  30  
 
INSIGHTS  MOCK  TEST  -­‐  31            
 

INSIGHTS ON INDIA MOCK PRELIMINARY EXAM - 2015

INSIGHTS ON INDIA MOCK TEST - 31


GENERAL STUDIES
PAPER-I
Time Allowed: 2 Hours Maximum Marks: 200

INSTRUCTIONS
1. IMMEDITELY AFTER THE COMMENCEMENT OF THE EXAMINATION, YOU SHOULD
CHECK THAT THIS TEST BOOKLET DOES NOT HAVE ANY UNPRINTED OR TORN OR MISSING
PAGES OR ITEMS, ETC. IF SO, GET IT REPLACED BY A COMPLETE TEST BOOKLET.
2. You have to enter your Roll Number on the Test
Booklet in the Box provided alongside. DO NOT
Write anything else on the Test Booklet.
4. This Test Booklet contains 100 items (questions). Each item is printed only in English. Each item
comprises four responses (answers). You will select the response which you want to mark on the
Answer Sheet. In case you feel that there is more than one correct response, mark the response which
you consider the best. In any case, choose ONLY ONE response for each item.
5. You have to mark all your responses ONLY on the separate Answer Sheet provided. See directions in
the Answer Sheet.
6. All items carry equal marks.
7. Before you proceed to mark in the Answer Sheet the response to various items in the Test Booklet, you
have to fill in some particulars in the Answer Sheet as per instructions sent to you with your
Admission Certificate.
8. After you have completed filling in all your responses on the Answer Sheet and the examination has
concluded, you should hand over to the Invigilator only the Answer Sheet. You are permitted to take
away with you the Test Booklet.
9. Sheets for rough work are appended in the Test Booklet at the end.
10. Penalty for wrong answers :
THERE WILL BE PENALTY FOR WRONG ANSWERS MARKED BY A CANDIDATE IN THE
OBJECTIVE TYPE QUESTION PAPERS.
(i) There are four alternatives for the answer to every question. For each question for which a
wrong answer has been given by the candidate, one-third of the marks assigned to that
question will be deducted as penalty.
(ii) If a candidate gives more than one answer, it will be treated as a wrong answer even if one of
the given answers happens to be correct and there will be same penalty as above to that
question.
(iii) If a question is left blank, i.e., no answer is given by the candidate, there will be no penalty
for that question.

http://www.insightsonindia.com

INSIGHTS ON INDIA MOCK TEST SERIES FOR CIVIL SERVICES PRELIMINARY EXAM 20

http://www.insightsonindia.com                                                          INSIGHTS   Page  1  


 
INSIGHTS  MOCK  TEST  -­‐  31            
 

3. The Reserve Bank of India (RBI)


recently signed a currency swap
1. What is/are the similarities between agreement with the Central Bank of Sri
phytoplankton in a pond ecosystem; and Lanka. This would help both the
lichens in a terrestrial ecosystem? countries in avoiding
1. They both can prepare their own a) Balance of payment and liquidity
food. crisis
2. They both are the pioneer species b) Low Inflation in the economies
forming the pioneer community. c) Adverse trade balance between both
3. They both are in a symbiotic nations
relationship with some species in d) Low cross-border capital inflows
their respective ecosystem.

Choose the correct answer using the codes


below. 4. Which of these are methods of ex-site
conservation?
a) 1 and 2 only 1. Cryopreservation
b) 1 and 3 only 2. Seed banks
c) 2 and 3 only 3. Field Gene banks
d) 2 only 4. Home gardens

2. Consider the following about Sickle Cell Choose the correct answer using the codes
Anaemia. below.
1. It is a hereditary disease a) 3 and 4 only
2. It spreads via air and water b) 1 and 2 only
3. It leads to leads to jaundice and c) 2, 3 and 4 only
frequent ailments in children d) All of the above
4. It can be prevented by increasing
iron intake since childhood

Choose the correct answer using the codes 5. A tiger reserve is generally zoned into
below. core, buffer and transition areas. In the
Core area, which of these activities are
a) 3 and 4 only NOT permitted in India?
b) 1 and 2 only 1. Tourism
c) 1 and 3 only 2. Mining
d) 2 and 4 only 3. Human habitation
4. Industries

http://www.insightsonindia.com                                                          INSIGHTS   Page  2  


 
INSIGHTS  MOCK  TEST  -­‐  31            
 
Choose the correct answer using the codes
below.
8. Marine algae have been used as manure
a) 1 and 3 only in many countries because
b) 2 and 4 only a) they contain very high biomass as
c) 2, 3 and 4 only compared to their weight
d) All of the above b) they are rich in nutrients
c) they help in decomposing the
detritus present in the farms
6. Susceptibility to extinction will be d) they produce agar that is highly
highest in species which fertile and keeps the soil moisture
1. have fixed migratory routes in fixed intact
seasons
2. have a narrow range of habitat
3. feed at lowest levels in the food web 9. How is social forestry different from
agro-forestry?
Choose the correct answer using the codes 1. In agro-forestry, trees are grown
below. instead of crops on farms.
a) 1 and 2 only 2. In social forestry, NGOs take
b) 1 and 3 only responsibility for afforestation of
c) 2 and 3 only arid and fallow lands.
d) All of the above 3. While the community owns the
resources in social forestry, agro-
7. Consider the following minerals and forestry produce is privately owned.
their uses. Choose the correct answer using the codes
1. Mercury is used in making electric below.
switches.
2. Phosphorous is used in making a) 1 and 2 only
medicines. b) 1 and 3 only
3. Potassium and Silver can be used in c) 2 only
photography. d) 3 only
4. Manganese can be used in making
disinfectants. 10. Consider the following statements about
the water cycle.
Choose the correct answer using the codes
below. Assertion (A): While evaporation is greater
over the oceans, rainfall is greater over the
a) 3 and 4 only land.
b) 1 and 2 only
c) 2, 3 and 4 only Reason (R): Trade winds carry moisture laden
d) All of the above winds from the oceans to land.

http://www.insightsonindia.com                                                          INSIGHTS   Page  3  


 
INSIGHTS  MOCK  TEST  -­‐  31            
 
In the context of the statements above, which 13. Which of the following is/are free-living
of these is true? nitrogen fixing bacteria present in the
soil?
a) A and R both are true, and R is the 1. Rhizobium
correct explanation for A. 2. Clostridium
b) A and R both are true, and R is the 3. Azotobacter
NOT the correct explanation for A. 4. Pseduomonas
c) A is correct, R is incorrect.
d) A is incorrect, R is correct. Choose the correct answer using the codes
below.

a) 1 and 4 only
11. Consider the following functions of b) 2, 3, and 4 only
forests. c) 2 and 3 only
1. Absorption of atmospheric gases d) 1, 2 and 3 only
2. Storage of minerals and water
3. Storage and release of radiant energy

These functions fall under which of the 14. Consider the following about
following categories? Mycorrhizae.
1. It is a symbiotic relation of fungi
a) Protective functions with plant roots.
b) Regulative functions 2. It helps in better absorption of
c) Productive Functions minerals from soil.
d) Reproductive Functions 3. It can form mutualistic relation with
blue-green algae to improve water
absorption in plants.
12. Which of these processes may lead to a
loss of nutrients from the soil? Choose the correct answer using the codes
1. Nutrient Immobilization below.
2. Mineralization a) 1 and 2 only
3. Leaching b) 1 and 3 only
4. Nitrification c) 2 only
Choose the correct answer using the codes d) 3 only
below.

a) 1 and 4 only 15. Certain plants use C4 pathway of


b) 2, 3, and 4 only photosynthesis instead of C3 generally
c) 2 and 3 only used by plants. What difference does a
d) 1, 2 and 3 only C4 pathway make to the plants?
1. It is a more efficient way of
photosynthesis.

http://www.insightsonindia.com                                                          INSIGHTS   Page  4  


 
INSIGHTS  MOCK  TEST  -­‐  31            
 
2. Plants can survive in low moisture b) 2 only
conditions. c) Both 1 and 2
3. Nutrient need from soil is lowered. d) None
4. They can survive in high temperature
conditions.

Choose the correct answer using the codes 18. Consider the following statements about
below. critical link species in a forest
ecosystem.
a) 1 and 2 only 1. A forest ecosystem has only a few
b) 1 and 3 only critical link species.
c) 2 and 4 only 2. Loss of critical link species has
d) All of the above disproportionate impact on the
ecosystem.
3. Critical link species are always found
16. Main inputs of phosphorous as a at the edge of ecosystems.
nutrient in soil come from 4. Critical link species are also known
1. Rainfall as Keystone species.
2. Phosphorous fixing bacteria Choose the correct answer using the codes
3. Weathering of soil below.
4. Atmospheric gas exchanges with soil
a) 1 and 2 only
Choose the correct answer using the codes b) 1 and 3 only
below. c) 2 and 4 only
a) 1 and 2 only d) 2 only
b) 1 and 3 only
c) 2 and 4 only
d) All of the above 19. During PM’s recent visit to Seoul, it was
announced that India and Korea will
strengthen their historic connection by
17. Only 50 per cent of the energy of the enhancing linkages of Korean people
total incident solar radiation is available with Ayodhya. What explains the
to producers for absorption. Why is the linkages?
other 50 per cent unavailable? a) Queen Suriratna, a princess from
1. It is lost in respiration and Ayodhya had travelled to South
assimilation by first level organisms. Korea to marry King Kim Suro
2. It is absorbed by gases or reflected by b) The land of Ayodhya was donated to
clouds in the atmosphere. the then King of South Korea as a
goodwill gesture who later returned
Which of the above is/are correct? the favour by gifting the land back

a) 1 only
http://www.insightsonindia.com                                                          INSIGHTS   Page  5  
 
INSIGHTS  MOCK  TEST  -­‐  31            
 
c) King Suro of South Korea helped the 2. Proroguing the Houses
Ayodhyan army to deal with invaders 3. Adjournment Sine dine
from the North-western mountains 4. Dissolving lower house
d) All of the above 5. Addressing both houses

Choose the correct answer using the codes


below.
20. Consider the following about the
Amaravati, Andhra Pradesh. a) 1, 4 and 5 only
1. It was ruled by Satavahanas. b) 2 and 3 only
2. Hinduism, Buddhism and Jainism c) 1, 2, 4 and 5 only
have all flourished in the region. d) All of the above
3. All major rock-cut architectural
schools were born in this region.

Choose the correct answer using the codes 23. The representative of each Union
below. territory in the Rajya Sabha are
indirectly elected by
a) 1 and 2 only a) Members of an electoral college
b) 1 and 3 only specially constituted for this purpose
c) 2 and 3 only b) Members of Legislative assemblies
d) All of the above c) Members of Local constitutional
bodies
21. Consider the following about Ken and d) MPs, MLAs and ward councillors
Betwa rivers. jointly from a Union Territory
1. Both are tributaries of Yamuna.
2. Both are major rivers of 24. A Member of Parliament ceases to hold
Bundelkhand region. his seat if
3. Both will be interlinked together as 1. He owns and runs a private
per a government decision. commercial firm
2. He holds any office of Profit under
Choose the correct answer using the codes the Union or State government
below. 3. He is removed from the political
a) 1 and 2 only party on whose ticket he was elected
b) 1 and 3 only 4. He is expelled from the house.
c) 2 and 3 only Choose the correct answer using the codes
d) 2 only below.

22. Which of the functions relating to the a) 2, 3 and 4 only


Parliament are performed by the b) 1 and 3 only
President? c) 2 and 4 only
1. Summoning the Houses d) All of the above

http://www.insightsonindia.com                                                          INSIGHTS   Page  6  


 
INSIGHTS  MOCK  TEST  -­‐  31            
 
c) Montague Chelmsford Act, 1919
d) Independence of India Act, 1947
25. The Speaker of Lok Sabha can be
removed on
1. A resolution passed by majority of
the membership of Lok Sabha 28. Rule of law in Article 14 of the
2. A resolution passed by special Constitution implies that
majority in Lok Sabha 1. The state cannot treat citizens
3. An enquiry by the Supreme Court differently with respect to the same
4. An order of the President law.
2. The state cannot punish citizens
Choose the correct answer using the codes except for violating the law.
below. 3. The state cannot confer any special
socio-economic privilege on any
a) 1 only individual.
b) 2 and 3 only
c) 1 and 3 only Choose the correct answer using the codes
d) 1, 3 and 4 only below.

a) 1 and 2 only
b) 2 and 3 only
26. Apart from the Supreme Court of India, c) 2 only
which of the following is/are authorities d) 1 only
empowered to interpret the
constitution?
1. Chief Justice, High Court
2. Speaker, Lok Sabha 29. Consider the following about Estimates
3. Chairman, Rajya Sabha Committee.
1. It is exclusively a committee of the
Choose the correct answer using the codes lower house.
below. 2. Chairman of this committee is
invariably from the ruling party.
a) 1 and 2 only 3. The committee is guided by
b) 2 and 3 only Comptroller and Auditor General of
c) 1 only India.
d) All of the above
Choose the correct answer using the codes
below.
27. The Institution of Speaker and Deputy a) 1 and 2 only
Speaker in India originated under the b) 2 and 3 only
provisions of c) 2 only
a) Charter Act 1891 d) 1 only
b) Morley-Minto Act, 1909

http://www.insightsonindia.com                                                          INSIGHTS   Page  7  


 
INSIGHTS  MOCK  TEST  -­‐  31            
 
30. Administration is held accountable to
the legislature by which of the following
devices? 32. What is the meaning of “popular
1. Financial Committees of the sovereignty”?
Parliament a) Views and preferences of the
2. Committee on delegated legislation majority (popular) are considered
3. Committee on Business advisory sovereign.
4. Question hour and zero hour in the b) A nation is sovereign in all respects
Parliament by virtue of the sovereignty of its
resident citizens.
Choose the correct answer using the codes c) An area can be declared sovereign by
below. the wish of the people.
d) People are the ultimate source of the
a) 1 and 3 only authority of their government.
b) 1, 2 and 4 only
c) 2 and 4 only
d) All of the above
33. The Source of power of “judicial review”
in the constitution flows from
a) The doctrine of separation of power
31. For a no-confidence motion to be between legislature and judiciary
admitted in Lok Sabha b) The authority of the Supreme Court
1. Speaker must give consent to it. and High courts to issue writs
2. Support of at least 50 members is (Article 32 and 226)
necessary. c) Article 13 of the Constitution
3. Leader of Opposition must lead the d) The Basic structure of the
motion. Constitution
4. Both houses of Parliament must be
in session.
5. Prior permission from the President
must be obtained. 34. Consider the following about All-India
services.
Choose the correct answer using the codes 1. Service conditions are determined by
below. the President.
2. Services can be abolished only by a
a) 1 and 2 only constitutional amendment.
b) 1, 2 and 5 only 3. UPSC is responsible for
c) 2, 3 and 4 only appointments to the posts in All-
d) 2 only India services.
4. Members of All-India services cannot
be removed from services by Prime
Minister alone.

http://www.insightsonindia.com                                                          INSIGHTS   Page  8  


 
INSIGHTS  MOCK  TEST  -­‐  31            
 
Choose the correct answer using the codes 37. Consider a situation where the
below. incumbent executive has been found
guilty of financial irregularities based on
a) 1, 2 and 4 only a CAG report as examined by the Public
b) 3 only Accounts Committee. The ultimate
c) 2, 3 and 4 only responsibility to take appropriate action
d) 2 and 4 only on this report lies on?
a) Prime Minister of India
b) President of India
35. In the Budgetary process, arrange the c) Parliament
following in correct order. d) Supreme Court
1. Vote on account
2. Bill sent to Upper House.
3. General Discussion on the Bill. 38. The International North–South
4. Bill referred to select committee. Transport Corridor will directly connect
Choose the correct answer using the codes India with which of these countries?
below. 1. Russia
2. Europe
a) 3142 3. Central Asia
b) 1423 4. Iran
c) 3412 5. Azerbaijan
d) 3421
Choose the correct answer using the codes
below.
36. Consider the following about the a) 2, 3 and 5 only
Economic Survey. b) 1, 3 and 4 only
1. It is published by the Ministry of c) 1 and 4 only
Finance. d) All of the above
2. It is presented along with the budget
to the Lok Sabha.
3. It is not presented to Rajya Sabha.
39. Consider the following about the project
Choose the correct answer using the codes SETI being led by Stephen Hawking?
below. 1. It will be a human space mission to
search for extra terrestrial
a) 1 and 2 only intelligence.
b) 3 only 2. The project will rely on sending
c) 2 and 3 only decrypted electromagnetic messages
d) 1 only in space.
3. The project has been undertaken in
joint collaboration with NASA.

http://www.insightsonindia.com                                                          INSIGHTS   Page  9  


 
INSIGHTS  MOCK  TEST  -­‐  31            
 
4. It is financial backed by the UK 3. East India Company had started
Government. interfering in the political matters of
Bengal.
Choose the correct answer using the codes 4. British did not pay appropriate
below. revenue to Bengal Nawab.
a) 1 only 5. East India Company did not stop
b) 2 and 3 only fortification in Bengal even after the
c) 1 and 4 only farman of Bengal Nawab.
d) None of the above Choose the correct answer using the codes
below.

40. After the death of Aurangzeb in 1707, a) 1, 2 and 3 only


Delhi could no longer function as an b) 4 and 5 only
effective centre. What can be the c) 3, 4 and 5 only
reason(s)? d) All of the above
1. Mughal governors (subedars)
asserted their authority over Mughal
Kingdoms. 42. Consider the following about Gandhian
2. Zamindars started establishing ideology?
regional kingdoms. 1. Gandhi was opposed to the existing
3. After his Death, British immediately state because it was forced and based
adopted the policy of ‘Paramountcy’. on centralization of authority.
2. As Per Gandhi, an all powerful state
Choose the correct answer using the codes impeded the progress of an
below. individual in society.
3. Gandhi held that ‘State’ represents
a) 1 and 2 only
violence in organized and
b) 1and 3 only
concentrated form.
c) 2 and 3 only
d) All of the above Choose the correct answer using the codes
below.

a) 1 and 2 only
41. The reason(s) behind Battle of Plassey
b) 2 and 3 only
between the Nawab of Bengal and the
c) 1 and 3 only
British was/were
d) All of the above
1. British denied stopping trade
relations with India
2. Bengal Nawab did not accept
‘subsidiary alliance’ enforced by the
British.

http://www.insightsonindia.com                                                          INSIGHTS   Page  10  


 
INSIGHTS  MOCK  TEST  -­‐  31            
 
43. Which of these is/are NOT principle(s) Reason (R): Suffering is caused by Ignorance
of United Nations? which is due to attachment, greed and
1. Some sovereigns are more equal than delusion.
others.
2. Member nations have rights but no In the context of the statements above, which
obligations. of these is true?
3. All members shall settle their a) A and R both are true, and R is the
international disputes by peaceful correct explanation for A.
means. b) A and R both are true, and R is the
4. All members shall settle their NOT the correct explanation for A.
national disputes by peaceful means. c) A is correct, R is incorrect.
Choose the correct answer using the codes d) A is incorrect, R is correct.
below.

a) 1 only 46. In Jainism, Abhayadana was related


b) 2 and 3 only with
c) 3 and 4 only a) Feeding the poor and homeless
d) 1, 2 and 4 only b) Imparting knowledge to the ignorant
c) Giving protection to someone under
threat
44. Which of the following concerning d) Selfless service given by the student
Wodeyar Dynasty is incorrect? to the teacher
a) After Tipu Sultan was killed in the
Battle of Seringapatam, Mysore was
placed under Wodeyars – the former
47. In the later Vedic period, Bhagadudha
ruling dynasty.
was
b) The kingdom was incorporated into
a) a local priest who performed
the Dominion of India after its
sacrifices for Brahmin households
independence from British rule.
b) an officer responsible for collection
c) Under British rule, Wodeyar Dynasty
of taxes
was exempted from Subsidiary
c) the village guard appointed by the
Alliance.
monarchy
d) None of the above
d) a jurist who settled civil disputes in
the village

45. Consider the following statements about


the teachings of Buddha.

Assertion (A): Every living being is in the state


of suffering and none is exception to this.

http://www.insightsonindia.com                                                          INSIGHTS   Page  11  


 
INSIGHTS  MOCK  TEST  -­‐  31            
 
48. Gandhi suspended which of these c) It is the first Indian book written on
movements calling it a ‘Himalayan the plight of transgender in ancient
Blunder’? India.
a) Non-cooperation movement d) None of the above
b) Satyagraha movement against
Rowlatt Act
c) Quit India movement 51. NASA’s New Horizons spacecraft was in
d) Civil Disobedience Movement news recently because
a) It is the first spacecraft to
successfully fly by Pluto.
49. Consider the following statements about b) It is first spacecraft to land safely on
Vaishnavism. an asteroid orbiting the Sun.
c) It recently escaped the solar system
Assertion (A): Its spread contributed to the to reach Kuiper belt.
growth of regional literature. d) It spotted an extremely dwarf black
Reason (R): It was adopted and followed by all hole in the solar system.
then contemporary regional poets and prose
writers.
52. The “rarest of the rare” principle in
In the context of the statements above, which handing out the death penalty was given
of these is true? by the Supreme Court in which case?
a) Shatrughan Chauhan vs Union of
a) A and R both are true, and R is the
India
correct explanation for A.
b) Bachan Singh vs State of Punjab
b) A and R both are true, and R is the NOT
c) Mithu vs State of Punjab
the correct explanation for A.
d) Ladani vs Union of India
c) A is correct, R is incorrect.
d) A is incorrect, R is correct.

53. Consider the following about the BRICS


New Development Bank.
50. The book 'One Part Woman' or
1. Agreement for establishing NDB was
Madhurobhagan was recently in news.
made at Fortaleza.
Why?
2. Bank will be headquartered in
a) Its writer has won the SAARC
Shanghai.
Literary Award recently.
3. Bank’s first President will be
b) The book was appreciated recently by
Kundapur Vaman Kamath from
the President of India for its depth
India.
on the issues afflicting transgender in
4. All member nations will have an
India.
equal say in Bank’s management.
5. All member nations will contribute
equally to Bank’s start-up capital.
http://www.insightsonindia.com                                                          INSIGHTS   Page  12  
 
INSIGHTS  MOCK  TEST  -­‐  31            
 
Choose the correct answer using the codes b) File a case in local police station
below. under Section 66A of the
Information Technology Act
a) 1 , 4 and 5 only c) File a case with the local police
b) 2 and 3 only station under the IPC Section
c) 1, 2, 3 and 4 only 499/500
d) All of the above d) Both (b) and (c)

54. According to a survey of trust in national 56. The 12th International Symposium on
governments for 2014 conducted by the Antarctic Earth Sciences (ISAES) was
Organisation for Economic Cooperation recently inaugurated at a function in
and Development (OECD), India has Goa. The theme(s) of the symposium
secured second position amongst 40 is/are
countries. According to the survey 1. Conservation strategies for
change in trust levels can be affected by? Antarctica
1. Economic outlook 2. Plate movements in Antarctica
2. Disasters 3. Human habitation in Antarctica
3. Major scandals
4. Election results Choose the correct answer using the codes
below.
Choose the correct answer using the codes
below. a) 1 and 2 only
b) 2 and 3 only
a) 1 and 4 only c) 1 and 3 only
b) 2 and 3 only d) 2 only
c) 1, 2 and 3 only
d) All of the above

57. As per the terms arrived at in the Iran


Nuclear Deal
55. You are a District Magistrate. You 1. Iran will not enrich any Uranium.
denied allotting a huge tender to an 2. Iran will be subject to oversight from
influential local businessman. To seek US Nuclear authorities.
revenge, he is posting defamatory and 3. All previous nuclear installations will
offensive speeches against you on the be shutdown in a period of time.
internet. Which legal recourse can you 4. Iran will not be able to share nuclear
take against him? technology with any nation.
a) Approach the State high court
accusing the businessman of abusing Choose the correct answer using the codes
freedom of speech given under below.
Article 19
a) 1 and 4 only

http://www.insightsonindia.com                                                          INSIGHTS   Page  13  


 
INSIGHTS  MOCK  TEST  -­‐  31            
 
b) 1 and 3 only 60. The government has decided to set up a
c) 2 and 3 only trade facilitation council. Consider the
d) None of the above following with reference to it.
1. The council will be chaired by the
Union Commerce and Industry
58. The Defence Acquisition Council (DAC) Minister.
that approves major defence related 2. It will be made a sub-organ of
projects consist of SAARC.
1. Minister of State for Defence 3. It will provide a single window
2. Cabinet Secretary mechanism for clearance of major
3. Chiefs of Army, Navy and Air Staff exports and imports.
4. Defence Secretary 4. It will include Ministers from states
5. Director, Intelligence Bureau also.
6. Director, RAW Choose the correct answer using the codes
7. National Security Advisor below.
Choose the correct answer using the codes a) 1 and 4 only
below. b) 1 and 3 only
a) 1, 2, 5, 6 and 7 only c) 2 and 3 only
b) 1, 3 and 4 only d) None of the above
c) 2, 3, 4 and 7 only
d) All of the above
61. Ottapayam situated in Kerala was
recently in news. Why?
59. While driving in an urban area, you find a) India’s first defence industrial park
out that the Air Quality Index (AQI) for will be situated here.
the area is “X” coloured. Air of this b) India’s biggest disabled friendly IT
quality may cause respiratory impact Park will be constructed here
even on healthy people, and serious supported entirely by World Bank.
health impacts on people with c) The first NIMZ in South India will be
lung/heart disease. Which colour is “X” built here.
most likely to be? d) It will host the biggest International
a) Light Pink Environmental Conservation
b) Yellow Conference held in India till date.
c) Maroon
d) Blue

http://www.insightsonindia.com                                                          INSIGHTS   Page  14  


 
INSIGHTS  MOCK  TEST  -­‐  31            
 
62. Consider the following with reference to 64. Jansankhya Sthirata Kosh” (JSK) or the
the National Pharmaceutical Pricing National Population Stabilisation Fund
Policy (NPPP). is under
1. Pharmaceutical companies cannot a) Ministry of Planning
revise the cost of drugs under the b) Ministry of Health and Family
policy without permission from the welfare
National Pharmaceutical Pricing c) Ministry of Statistics and Program
Authority. implementation
2. It is applicable to all drugs on the d) None of the above
national list on life-saving medicines.
3. Drug pricing in the policy is fixed
based on a formula. 65. As per the Antarctica treaty
Which of the above is/are correct? 1. No commercial activity is allowed in
Antarctica.
a) 1 and 2 only 2. Each member nation will adopt one
b) 2 and 3 only of Antarctica’s endangered species.
c) 1 and 3 only 3. All claims to the Antarctica territory
d) All of the above are void.
4. Any scientific study needs the
approval of Scientific Committee on
63. A geographical indication (GI) is a name Antarctic Research (SCAR).
or sign used on certain products which Choose the correct answer using the codes
corresponds to a specific geographical below.
location or origin. Which of these are
GIs in India? a) 1 and 3 only
1. Mysore Silk b) 2 and 4 only
2. Darjeeling Tea c) 1, 3 and 4 only
3. Kancheepuram Saree d) All of the above
4. Thanjavur Paintings

Choose the correct answer using the codes


below. 66. As compared to the unamended
principal land Acquisition act passed in
a) 1 and 4 only 2013, the Right to Fair Compensation
b) 1 and 3 only and Transparency in Land Acquisition,
c) 2 and 3 only Rehabilitation and Resettlement
d) 1, 2 and 4 only (Amendment) Ordinance, 2015 relaxes
provisions relating to which of the
following categories?
1. Social Impact assessment

http://www.insightsonindia.com                                                          INSIGHTS   Page  15  


 
INSIGHTS  MOCK  TEST  -­‐  31            
 
2. Restrictions on acquisition of multi- 4. It is a UNESCO World Heritage Site.
cropped land 5. The temple was originally built at the
3. Consent for public private mouth of the river Chandrabhaga.
partnerships (PPPs) projects
Choose the correct answer using the codes
Choose the correct answer using the codes below.
below.
a) 2, 3 and 5 only
a) 1 and 2 only b) 1, 2, 3 and 4 only
b) 2 and 3 only c) 1, 4 and 5 only
c) 1 and 3 only d) All of the above
d) All of the above
69. The Basilica of Bom Jesus located in
Goa, India is an important cultural
67. Consider the following about the Stone heritage because
chariot at Vittala Temple, Hampi. a) It is the oldest church in World.
1. It is made of granite. b) It contains the mortal remains of St.
2. An image of Garuda was originally Francis.
enshrined within its sanctum. c) It was constructed jointly by all
3. It is a monolithic structure. colonial powers that came to India.
4. Its base platform is carved with d) While the exterior is baroque
mythical battle scenes. architecture, interior is highly
5. Two horses are positioned before the contrastingly Hindu and Mughal
chariot as if they are pulling it. architecture which is nowhere found
in the world.
Choose the correct answer using the codes
below. 70. The famous Padmapani painting in the
Ajanta Caves depicts
a) 2, 3 and 5 only
a) a bodhisattva with a blue lotus flower
b) 1, 2, 3 and 4 only
sitting among a crowd of devotees
c) 1, 2 and 4 only
b) a feminine Buddhist Sangha being
d) 1, 3 and 5 only
ordained by Gautama Buddha
c) Buddha’s Mahaparinirvana being
witnessed by his very close disciples
68. Consider the following about Sun d) Buddha conquering Mara of all
Temple, Konark. worldly passions
1. It is an example of Kalinga
architecture. 71. Cloudbursts can be caused by
2. Temple was designed as a gigantic 1. Excess of electrostatic induction in
chariot of Sun God. the clouds
3. The alignment of the Sun Temple is 2. High jet stream velocity in the upper
on the east-west direction. atmosphere
http://www.insightsonindia.com                                                          INSIGHTS   Page  16  
 
INSIGHTS  MOCK  TEST  -­‐  31            
 
3. Mixing of warm air parcel with Choose the correct answer using the codes
cooler air resulting in sudden below.
condensation
a) 1 and 2 only
Choose the correct answer using the codes b) 2 and 3 only
below. c) 1 and 3 only
d) All of the above
a) 1 and 2 only
b) 2 and 3 only 74. Presently space activities in India are
c) 1 and 3 only guided by
d) All of the above 1. Satellite Communications Policy,
2000
72. The Karnataka High Court has asked the 2. International Agreements
Karnataka Forest department to 3. Revised Remote Sensing Policy 2011
consider banning eucalyptus plantations
across the state. Why? Choose the correct answer using the codes
a) It is not natural to the Karnataka below.
region and is affecting the
biodiversity of the region. a) 1 and 2 only
b) Tall eucalyptus trees do not allow b) 2 and 3 only
sufficient sunlight and nutrition to c) 1 and 3 only
other forest trees. d) All of the above
c) Large area of forest was already
replaced with monoculture 75. What is/are the benefit(s) of having a
eucalyptus and planting more trees separate and unified National
would affect the species diversity of Renewable Energy Act (NERA)?
the region. 1. The Ministry of New and renewable
d) The eucalyptus tree consumes a lot of energy need not depend on other
water for its growth depleting ministries for necessary clearances.
groundwater levels. 2. The law will clarify institutional
mechanisms related with financing,
73. Consider the following about GSLV monitoring, planning and support
Mark III developed by ISRO. for renewable energy generation.
1. It features an Indian cryogenic Which of the above is/are correct?
technology.
2. It can launch communication a) 1 only
satellites in space. b) 2 only
3. It is presently being used for c) Both 1 and 2
commercial satellite launches by d) None
ISRO.

http://www.insightsonindia.com                                                          INSIGHTS   Page  17  


 
INSIGHTS  MOCK  TEST  -­‐  31            
 
76. The “collective self-defence” doctrine b) Fold Mountains will be created in
presently being pushed by the Japanese both the cases.
government means c) Fold Mountains may be created in
a) Japan is not in favour of independent case 1 and volcanic mountains in
armies for all nations case 2.
b) Japan may wish to send troops d) Volcanic Mountains may be created
abroad to rescue allies under attack in case 1 and Fold Mountains in case
c) Japan will intervene constructively in 2.
all international negotiations that
have significant geo-strategic 79. Most of the rivers in Africa cannot be
implications navigated fully. Why?
d) Japan will make military a) They follow highly tortuous and
conscription compulsory for its narrow beds.
citizens in larger national interest b) A large number of rivers flow
through narrow rift valleys.
77. Why fossils are generally found in low- c) They form waterfalls.
lying areas? d) River flow in most of the rivers is too
a) Faulting and folding of earth’s crust less to become navigable.
puts low-lying areas under very high
temperature and pressure
b) Low-lying areas are at a higher 80.Atmosphere is heated mainly from
temperature than areas of higher 1. Solar insolation reflected back to
altitude, hence more organic matter upper troposphere by clouds
is converted to fossil fuels 2. Convection from heated surface of
c) Deposition of sediments is carried by the earth
rivers that also carry organic matter 3. Direct solar radiation
d) Volcanic magma spreads only on
low-lying areas which is a significant Choose the correct answer using the codes
source of fossils below.

a) 1 and 2 only
78. Consider two geo-morphological cases. b) 2 and 3 only
1. A continental plate colliding with an c) 1 and 3 only
oceanic plate and the latter subsiding d) 2 only
below the former.
2. Two continental plates colliding.

Which of these outcomes is possible in both the 81. Consider the following about Katabatic
cases? winds.
1. They are generally experienced
a) Volcanic mountains may be created during winter.
in both the cases. 2. They blow adjacent to Highlands.

http://www.insightsonindia.com                                                          INSIGHTS   Page  18  


 
INSIGHTS  MOCK  TEST  -­‐  31            
 
3. These winds may increase the
temperature and pressure of an area.
4. The winds are very dry. 84. Consider the following statements.
1. All the nine planets (including Pluto)
Choose the correct answer using the codes of the solar system revolve around
below. the Sun in counter-clockwise
direction.
a) 1 and 4 only 2. All planets in the solar system rotate
b) 2 and 3 only in the same direction in which they
c) 1 and 3 only revolve.
d) All of the above
Which of the above is/are true?

a) 1 only
82. How are tides a great aid in shipping b) 2 only
and navigation? c) Both 1 and 2
1. High tide increases the depth of d) None
water near the coasts.
2. They take away the mud brought by
rivers preventing silting of harbours.
3. They make the coastline indented 85. Why is it that we always see one side of
which supports natural harbours. the moon while the other side always
remains away from us?
Choose the correct answer using the codes a) Relative positions of Moon in Earth’s
below. orbit around the Sun form a circle.
b) Moon takes the same time to revolve
a) 1 and 2 only around the earth and rotate on its
b) 2 and 3 only own axis.
c) 1 and 3 only c) Frequent eclipsing of the Moon by
d) All of the above the Sun and other planets
d) None of the above

83. Some ports of even Polar Regions


remain ice-free during winter. This is 86. Consider the following about latitudes
because of the and longitudes.
a) Influence of Warm currents across 1. Equator and Prime meridian are the
the coasts largest circles that can be drawn on
b) Global warming and its effect on earth.
albedo of earth 2. All meridians are of equal length.
c) High salt content in these oceans 3. Time zone of a place is determined
that prevent freezing and thawing both by longitude and latitude.
d) Dense fog and mist in these regions
that trap solar heat

http://www.insightsonindia.com                                                          INSIGHTS   Page  19  


 
INSIGHTS  MOCK  TEST  -­‐  31            
 
Choose the correct answer using the codes b) may show an unclear relationship of
below. quantity consumed with rising
consumer income
a) 1 and 2 only c) are consumed in more quantity
b) 2 and 3 only because they are in greater demand
c) 1 and 3 only in public
d) 2 only d) are in greater demand because they
are in scarcity

87. Vertical sun rays have a greater heating


effect as compared to oblique rays 90. In Economic theory and policy making,
because ‘Consumer Welfare’ is deemed to be
1. They cover shorter distance in maximized when
atmosphere to reach earth. a) every individual is able to purchase
2. They are spread over a larger area on goods at the exact market price they
earth. were willing to pay
Which of the above is/are correct? b) a large number of individuals are
able to purchase goods at a much
a) 1 only lower market price that what they
b) 2 only were willing to pay
c) Both 1 and 2 c) all individuals attain an ideal state of
d) None having no demand for any more
goods and services
d) they are able to purchase goods
beyond their means of earning

88. Who decides what images appear in the


currency notes that are printed by
91. Real Gross Domestic Product (GDP) of a
Reserve Bank of India for money supply
country will rise if
in the economy?
1. There is high inflation in the
a) Union Minister of Finance
economy but no growth in
b) RBI Governor
production
c) Director-General of Archaeological
2. There is high growth in production
Survey of India
but no inflation in the economy
d) None of the above
3. There is very little growth in
production but heavy deflation in the
economy
89. Giffen goods are those goods that 4. There is very little deflation in the
a) may show an inverse relationship of economy but no growth in
quantity consumed with rising production
consumer income
http://www.insightsonindia.com                                                          INSIGHTS   Page  20  
 
INSIGHTS  MOCK  TEST  -­‐  31            
 
Choose the correct answer using the codes b) Foreign Exchange reserves
below. c) Bank deposits of Public
d) Revenues of Government
a) 1 and 4 only
b) 2 and 3 only 95. The situation of liquidity trap occurs in
c) 1 and 3 only the economy
d) 2 and 4 only 1. When foreign exchange reserves dry
up.
92. How are employment and interest rates 2. When Bank deposits are low so as to
related in an economy? cause a Bank default on its
a) Generally higher interest rates customers
impede a higher growth in 3. When interest rates are very low so
employment numbers. that everyone anticipates a rise in
b) Generally lower interest rates impede interest rates
a higher growth in employment 4. When people don’t wish to hold any
numbers. bonds
c) There is no relationship of interest
rate and employment in an economy. Choose the correct answer using the codes
d) Depending on the type of economy, below.
any of the statements (a), (b) or (c)
can be true. a) 1 and 4 only
b) 2 and 3 only
93. The Reserve Bank of India draws its c) 1 and 3 only
authority from d) 3 and 4 only
1. Constitution of India
2. Parliamentary Law 96. The main objective(s) of the Public
3. Conventions and Customs Distribution System (PDS) was/were to
4. Executive Council of Ministers 1. Ensure optimum export of
agricultural commodities
Choose the correct answer using the codes 2. To prevent any import of agricultural
below. produce
3. To ensure food security
a) 1 and 4 only 4. To ensure healthy meals are taken by
b) 2 only each and every family in India
c) 1 and 3 only
d) 1, 2 and 3 only Choose the correct answer using the codes
below.

94. Sterilization operation is undertaken by a) 1, 2 and 4 only


the Central bank of a country to prevent b) 2 and 3 only
a shock mainly to the c) 3 only
a) Money supply in the economy d) 3 and 4 only

http://www.insightsonindia.com                                                          INSIGHTS   Page  21  


 
INSIGHTS  MOCK  TEST  -­‐  31            
 
b) GDP structure is changing to favour
the Services sector more than the
97. Consider the following statements. industrial sector
Assertion (A): Devaluation of currency may c) Foreign capital inflows in the
lead to improvement in trade balance. economy are not as strong as
domestic investment
Reason (R): Devaluation of currency leads to d) Financial sector in the country is
more competitive export prices. becoming increasingly globalized and
risky
In the context of the statements above, which
of these is true?

a) A and R both are true, and R is the 100. With reference to The Energy and
correct explanation for A. Resources Institute, consider the
b) A and R both are true, and R is the following statements:
NOT the correct explanation for A. 1. It was formerly known as Tata
c) A is correct, R is incorrect. Energy Research Institute
d) A is incorrect, R is correct. 2. It is an autonomous research
institution which works under the
Ministry of Renewable and Non-
98. In which of the following situations an Renewable Energy Sources,
expansion in the money supply may not Gvernment of India
lead to inflation?
Which of the above statements is/are correct?
1. When economy is operating at a
much lower level of production than a) 1 Only
its capacity b) 2 Only
2. When supply of goods meets the c) Both
demands posed by the extra money d) None
supply

Which of the above is/are correct?

a) 1 only
b) 2 only  
c) Both 1 and 2
d) None

99. A country will NOT develop


economically if
a) Increasingly larger share of the GDP
is coming from the Primary sector

http://www.insightsonindia.com                                                          INSIGHTS   Page  22  


 

You might also like